You are on page 1of 6

Solutions to Exam 3, Math 10560

sin n
1. Compute the following limit: lim .
n→∞ n2
−1 sin n 1 −1 1
Solution: Note 2 ≤ 2 ≤ 2 . Both 2 and 2 tend to zero as n tends to
n n n n n
infinity. So by taking the limits of the bounding functions and using the Squeeze
sin n
Theorem, we get lim = 0.
n→∞ n2

3n2 (n − 2)!
2. Compute the following limit lim .
n→∞ n!
Solution: Note
3n2 (n − 2)! 3n2 (n − 2)! 3n2 3n
= = =
n! n(n − 1)(n − 2)! n(n − 1) n−1
so
3n2 (n − 2)! 3n 3
lim = lim = lim 1 = 3.
n→∞ n! n→∞ n − 1 n→∞ 1 −
n


X 3 + 2n
3. Does the series converge or diverge? If it converges, compute its value.
π n+1
n=0
Solution: Note

X ∞ ∞ µ ¶ ∞ ∞
3 + 2n 1 X 3 + 2n 1X 3 2n 1X 3 1 X 2n
= = + = + .
π n+1 π πn π πn πn π πn π πn
n=0 n=0 n=0 n=0 n=0

The two sums are both geometric series, the first with a = 3 and |r| = |1/π| < 1
and the second with a = 1 and |r| = |2/π| < 1. Hence each series converges and
our splitting the series into two was valid. Moreover
∞ ∞
à ! à !
1X 3 1 X 2n 1 3 1 1 3 1
n
+ n
= 1 + 2 = + .
π π π π π 1− π π 1− π π−1 π−2
n=0 n=0

P

n2 +1√
4. Which of the following statements are true about the series n5 −n2 3
?
n=1

n2 + 1
I. This series converges because lim √ = 0.
n→∞ n5 − n2 3
II. This series converges by Ratio Test.

X 1
III. This series converges by Limit Comparison Test against the p-series .
n3
n=1

1
Solution: Look at each part.

n2 + 1
I. Although lim √ = 0, we cannot conclude anything from this. (This
n→∞ n5 − n2 3
is using the Test for Divergence which is inconclusive here.)
¯ ¯ ¯
¯ ¯ ¯ (n+1)2 +1 ¯ ¯ n2 + 2n + 2 √ ¯
5 − n2 3 ¯
¯ an+1 ¯ ¯ (n+1)5 −(n+1)2 √3 ¯ ¯ n ¯
II. lim ¯¯ ¯ = lim ¯ ¯ = lim ¯
¯ n→∞ ¯ n5 + · · · + 1 − √3 · n2 + 1 ¯¯
n→∞ an ¯ n→∞ ¯¯ n2 +1√
¯
¯ 7 n5 −n2 3
√ ¯
¯ 2 ¯
= lim ¯ nn7+···−2n √ 3 ¯ = 1; this is the one situation in which the Ratio Test is
+···+1− 3
n→∞
inconclusive.
√ ∞
1
n5 − n2 3 X 1
n3
III. lim n2 +1 = lim 5 3
= 1, so by (limit) comparison with ,
n→∞ √ n→∞ n + n n3
5
n −n 32 n=1
the series converges.

Therefore only III is true.


X cos(2n)
5. One of the statements below holds for the series . Which one?
n2 + 1
n=1
(a) This series is absolutely convergent by Comparison Test.
(b) This series is conditionally convergent.
(c) This series converges by Alternating Series Test.
(d) This series diverges by Ratio Test.
(e) This series diverges because lim cos(2n)
n2 +1
is not 0.
n→∞
¯ ¯ ¯ ¯
¯ 1 ¯¯ 1 ¯ cos(2n) ¯ 1
Solution: Note | cos(2n)| < 1 and ¯¯ ≤ for all n; thus ¯ ¯
¯ n2 + 1 ¯ ≤ n2 for
n2 + 1 ¯ n2
X∞ X∞ ¯ ¯
1 ¯ cos(2n) ¯
all n. Since converges as a p-series with p = 2 > 1, so does ¯ ¯
n2 ¯ n2 + 1 ¯
n=1 n=1
by the Comparison Test. So the original series converges absolutely, and (a) is true.
The remaining statements are false: (b), because if the series were conditionally
convergent it would not be absolutely convergent; (c), because although the series
has positive and negative terms, it is not alternating, so the alternating series test
does not apply; (d), because the ratio test leads to a limit of 1, which is inclusive,
and (e), because the series converges, so the limit as n tends to infinity of the nth
term is 0.


X (−1)n−1
6. Which of the following statements are true about the series ?
n2
n=1

I. This series converges by the Alternating Series Test.


II. This series converges by the Ratio Test.
III. This series converges absolutely.

Solution:

2
1 1
I. 2
is decreasing and lim 2 = 0 so the Alternating Series Test says that
n n→∞ n
X∞
(−1)n−1
converges.
n2
n=1
¯ ¯ ¯ 1 ¯ ¯ ¯
¯ an+1 ¯ ¯ ¯ ¯ n2 ¯
¯ ¯ ¯ (n+1)2 ¯ ¯ ¯ = 1, so we get no conclusion
II. lim ¯ = lim ¯ 1 ¯ = lim ¯
n→∞ an ¯ n→∞ ¯
n 2 ¯ n→∞ (n + 1)2 ¯
from the Ratio Test.
(−1)n−1
III. Taking the absolute value of results in a p-series with p = 2 > 1, so
n2
we conclude that the series converges absolutely.
So I and III are true, II is false.


X
7. Compute the radius of convergence of the power series 2n (x − 1)2n .
n=1
p
Solution: Using the Root Test lim n |2n (x − 1)2n | = lim |2(x − 1)2 | = |2(x −
n→∞ n→∞
√ √
2 2 2 1 2 2
1) |. Now |2(x − 1) | < 1 ⇒ (|x − 1|) < ⇒ |x − 1| < . So R = .
2 2 2
The same conclusion could be reached using the ratio test.

π
8. Identify the Taylor Series of f (x) = sin(x) centered at a = 2 and its interval of
convergence.
Solution: Note

f (x) = sin x, f 0 (x) = cos x, f 00 (x) = − sin x, f 000 (x) = − cos x, f 0000 (x) = sin x, ...

so
π π π π
f ( ) = 1, f 0 ( ) = 0, f 00 ( ) = −1, f 000 ( ) = 0, ...
2 2 2 2
(with the pattern 1, 0, −1, 0 repeating).
So the Taylor Series is:
(x − π2 )2 (x − π2 )4 (x − π2 )6
1− + − + ...
2! 4! 6!

X (−1)n (x − π2 )2n
which in summation notation is .
(2n)!
n=0
Since
¯ ¯
¯ ¯ ¯ (−1)n+1 (x− π2 )2(n+1) ¯ ¯ ¯
¯ an+1 ¯
¯ ¯
¯
¯ (2(n+1))! ¯
¯
¯
¯ (x − π2 )2 ¯
¯
lim ¯ = lim ¯ n→∞ ¯ (2n + 2)(2n + 1) ¯ = 0
= lim
an ¯ n→∞ ¯¯ (−1) (x− 2 )
n π 2n
n→∞ ¯
(2n)!

regardless of the value of x, from the Ratio Test we conclude that the series con-
verges for all x (interval of convergence is (−∞, ∞)).

3
9. The following is the fourth order Taylor polynomial of the function f (x) at a.
√ 1
T4 (x) = 10 + 5(x − a) + 3(x − a)2 + (x − a)3 + 17e(x − a)4

What is f 000 (a)?
f 000 (a) 1
Solution: By the Taylor formula, we have = (which is the coefficient
3! 2π
1·2·3 3
of (x − a)3 ) and hence f 000 (a) = = .
2π π

2
10. a) (5 pts) Give a power series representation for ex .
Solution: Since the n-th derivative of ex is ex and e0 = 1,

X ∞
X
f (n) (0) 1 n
ex = xn = x .
n! n!
n=0 n=0

Substituting x2 for x, we have



X
x2 1 2n 1
e = x = 1 + x2 + x4 + · · · .
n! 2
n=0

b) (5 pts) Find the limit


2
ex − 1 − x2
lim .
x→0 x4

Solution:
2
ex − 1 − x 2 (1 + x2 + 21 x4 + 16 x6 + · · · ) − 1 − x2 1 4 1 6
2x + 6x + · · · 1 1
4
= 4
= 4
= + x2 +· · · .
x x x 2 6
So 2
ex − 1 − x2 1 1 1
lim 4
= lim + x2 + · · · = .
x→0 x x→0 2 6 2
(This part could also be done using L’Hospital’s rule, but it would require multi-
2 2 2 2
ex −1−x2 2xex −2x ex −1 2xex
ple iterations: limx→0 x4
= limx→0 4x3
= limx→0 2x2
= limx→0 4x =
x 2
limx→0 e2 = 1
2. )

1
11. Consider the function f (x) = 2−3x .

a) (4 pts.) Find the Taylor series of f (x) centered at 0.


Solution: We have
à ! ∞ ∞
1 1 1 1 1X 3 X 3n
= = = ( x)n = xn ,
2 − 3x 2(1 − 32 x) 2 1 − 32 x 2
n=0
2 2n+1
n=0

as long as |r| = | 32 x| < 1.

4
b) (3 pts.) Determine the radius of convergence of this power series.
Solution: From our knowledge of geometric series, the series converges if and
2
only if |r| = | 23 x| < 1, which implies |x| < 23 , and so the radius R is R = . One
3
can also arrive at this conclusion by using the Ratio Test or the Root Test.
1
c) (4 pts) Find a power series representation for (2−3x) 2 and give its radius of
convergence.
Solution: Differentiating the function in part a) and differentiating term by
term the corresponding power series representation found in part a), we get

X n3n
3
= xn−1 .
(2 − 3x)2 2n+1
n=1
Dividing both sides by 3, we get

X n3n−1 ∞
X (n + 1)3n
1 n−1
= x = xn .
(2 − 3x)2 2n+1 2n+2
n=1 n=0

The radius of convergence of this series is 2/3, the same as that of the series in
part a).
You can also obtain the answer by squaring the power series found in part a):
1 3 3 2 3 3 3 3 2 3 3 1 3
4 (1 + 2 x + ( 2 x) + ( 2 x) + · · · )(1 + 2 x + ( 2 x) + ( 2 x) + · · · ) = 4 (1 + 2( 2 x) +
3 2 3 3
3( 2 x) + 4( 2 x) + · · · ).
d) (1 pt) What is the value of the series you found in part (c) at x = 1/2?
Solution: The value is given by
¯
1 ¯ 1
¯ = = 4.
(2 − 3x) 2 ¯ 1 (2 − 3( 1 ))2
x= 2 2

12. Find the interval of convergence of the following power series:



X (x + 1)n
.
n
n=1

Solution: Using the Ratio Test


¯ (x+1)n+1 ¯
¯ ¯ n
¯ n+1 ¯
lim ¯ (x+1) n ¯ = lim |x + 1| = |x + 1| .
n→∞ ¯ ¯ n→∞ n + 1
n
We want this to be less than one. So −1 < x + 1 < 1 which implies −2 < x < 0.
Next, we need to check the end points.
X∞
(−1)n
• x = −2 : We have which converges by the Alternating Series Test.
n
n=1

X (1)n
• x = 0 : We have which diverges because it is a p-series with p = 1.
n
n=1
Hence, the interval of convergence is [−2, 0).

5
13. Use the Integral Test to determine whether the series

X ln(n)
n3
n=2

is divergent or convergent. You must show that the Integral Test can be used in
this situation.
Note: A correct answer with no work is worth only 3 points.
Hint: Use Integration By Parts.
µ ¶
d ln(x) 1 − 3 ln(x)
Solution: Note 3
= . This will be negative when the numer-
dx x x4
ator is negative, that is when
1 √
0 > 1 − 3 ln(x) ⇔ 3 ln(x) > 1 ⇔ ln(x) > ⇔ x > 3 e.
3
√ ln(x)
This holds for x ≥ 2, since 8 > e or 2 > 3
e. Therefore, is positive, decreasing
x3
and continuous (since
Z ∞it is differentiable) for x ≥ 2. We can use the Integral Test.
ln(x) 1 1
Next, we evaluate 3
dx. Let u = ln(x) and dv = 3 dx. Then du = dx
2 x x x
x−2
and v = . Using Integration by Parts, we get
−2
Z Z
ln(x) − ln x 1 ln x 1
3
dx = 2
+ 3
dx = − 2 − 2 + C.
x 2x 2x 2x 4x
So using the definition of an improper integral and L’Hôpital’s Rule
Z ∞ µ ¶ Ã !
1
ln(x) ln t 1 ln 2 1 ln 2 1
3
dx = lim − 2 − 2 + + = lim − t − 0 + +
2 x t→∞ 2t 4t 8 16 t→∞ 4t 8 16
ln 2 1
= + .
8 16
The integral converges, therefore the series converges.

You might also like